Đến nội dung

NTA1907 nội dung

Có 1000 mục bởi NTA1907 (Tìm giới hạn từ 03-05-2020)



Sắp theo                Sắp xếp  

#623732 Topic về Bất đẳng thức, cực trị THCS

Đã gửi bởi NTA1907 on 30-03-2016 - 21:33 trong Bất đẳng thức và cực trị

Cho a,b,c>0.t/m a+b+c=3

Chứng minh:

 

$\frac{a+1}{b^{2}+1}+\frac{b+1}{c^{2}+1}+\frac{c+1}{a^{2}+1}\geq 3$ 

                                 ( hsg tỉnh Nghệ An 2016)

Áp dụng kĩ thuật AM-GM ngược ta có:

$a-\frac{a+1}{b^{2}+1}=\frac{ab^{2}-1}{b^{2}+1}\leq \frac{ab^{2}-1}{2b}=\frac{ab}{2}-\frac{1}{2b}$

$\Leftrightarrow \frac{a+1}{b^{2}+1}\geq a-\frac{ab}{2}+\frac{1}{2b}$

$\Rightarrow \sum \frac{a+1}{b^{2}+1}\geq \sum a-\sum \frac{ab}{2}+\sum \frac{1}{2b}\geq 3-\frac{(a+b+c)^{2}}{6}+\frac{1}{2}.\frac{9}{a+b+c}=3$

Dấu = xảy ra$\Leftrightarrow a=b=c=1$




#605914 Topic về Bất đẳng thức, cực trị THCS

Đã gửi bởi NTA1907 on 29-12-2015 - 12:58 trong Bất đẳng thức và cực trị

Cho a,b,c là các số thực dương thỏa mãn $(a+b)(b+c)(c+a)=1$. Chứng minh rằng:  $ab+bc+ca\leq \frac{3}{4}$

Ta có:

$(a+b)(b+c)(c+a)=(a+b+c)(ab+bc+ca)-abc$

Mà $(a+b+c)(ab+bc+ca)\geq 3\sqrt[3]{abc}.3\sqrt[3]{(abc)^{2}}=9abc$

$\Rightarrow 1\geq (a+b+c)(ab+bc+ca)-\frac{1}{9}(a+b+c)(ab+bc+ca)$

$\Leftrightarrow \frac{81}{64}\geq (a+b+c)^{2}(ab+bc+ca)^{2}\geq 3(ab+bc+ca)^{3}$

$\Rightarrow ab+bc+ca\leq \frac{3}{4}$




#608084 Topic về Bất đẳng thức, cực trị THCS

Đã gửi bởi NTA1907 on 08-01-2016 - 22:16 trong Bất đẳng thức và cực trị

Biết $\sum x^{2}=3$

Chứng minh $\sum \frac{xy}{z}\geq 3$

đây




#632975 Topic về Bất đẳng thức, cực trị THCS

Đã gửi bởi NTA1907 on 13-05-2016 - 22:35 trong Bất đẳng thức và cực trị

Chứng minh rằng $\frac{a+b}{\sqrt{a(3a+b)}+\sqrt{b(3b+a)}}\geq\frac{1}{2}$

Ai giúp mình với mình đang cần lắm

Mình cảm ơn trước nhé

Áp dụng Cauchy-Schwarz ta có:

$\frac{a+b}{\sqrt{a(3a+b)}+\sqrt{b(3b+a)}}\geq \frac{a+b}{\sqrt{(a+b)(3a+b+3b+a)}}=\frac{1}{2}$




#607945 Topic về Bất đẳng thức, cực trị THCS

Đã gửi bởi NTA1907 on 08-01-2016 - 13:16 trong Bất đẳng thức và cực trị

Cho a,b,c là các số dương.Tìm MAX của:

$\frac{\sqrt{ab}}{a+b+2c}+\frac{\sqrt{bc}}{b+c+2a}+\frac{\sqrt{ca}}{c+a+2b}$

Khi thay $(\sqrt{a},\sqrt{b},\sqrt{c})\rightarrow (a,b,c)$ thì bài toán trên tương tự bài toán sau:

918a28593aab437fc1f9a67bab5e2feaf2db6d46.jpg

d5389607c001daec91cb4436a4050bc38ebc3df9.jpg




#607734 Topic về Bất đẳng thức, cực trị THCS

Đã gửi bởi NTA1907 on 07-01-2016 - 13:09 trong Bất đẳng thức và cực trị

ai giải bài này đi

Ta có:
$a^{2}+2b+3=(a^{2}+1)+2b+2\geq 2(a+b+1)$
$\Rightarrow \sum \frac{a}{a^{2}+2b+3}\leq \sum \frac{a}{2(a+b+1)}\leq \frac{1}{2}$
Ta cm: $\sum \frac{a}{a+b+1}\leq 1$
$\Leftrightarrow \sum (\frac{a}{a+b+1}+\frac{b+1}{a+b+1})\leq 1+\sum \frac{b+1}{a+b+1}$
$\Leftrightarrow \sum \frac{b+1}{a+b+1}\geq 2$
Ta có:
$\sum \frac{b+1}{a+b+1}=\sum \frac{(b+1)^{2}}{(b+1)(a+b+1)}\geq \frac{(a+b+c+3)^{2}}{\sum (b+1)(a+b+1)}=2$(đpcm)
Dấu = xảy ra$\Leftrightarrow a=b=c=1$



#632382 Inequalities From 2016 Mathematical Olympiads

Đã gửi bởi NTA1907 on 10-05-2016 - 22:31 trong Bất đẳng thức - Cực trị

Bài 32. (Kyiv Mathematical Festival)

1. Cho ba số thực không âm $a,b,c$ thỏa mãn điều kiện $ab+bc+ca=2.$ Chứng minh rằng \[\frac{ab}{c+1}+\frac{bc}{a+1}+\frac{ca}{b+1}+2(a+b+c) \geqslant 6.\]

2. Cho ba số thực không âm $a,b,c$ thỏa mãn điều kiện $ab+bc+ca=3.$ Chứng minh rằng \[\frac{ab}{c+1}+\frac{bc}{a+1}+\frac{ca}{b+1} \geqslant \frac{3}{2}.\]

2. Áp dụng Cauchy-Schwarz ta có:

$\sum \frac{(ab)^{2}}{abc+ab}\geq \frac{(ab+bc+ca)^{2}}{3abc+ab+bc+ca}=\frac{3}{abc+1}\geq \frac{3}{2}$(vì $abc\leq 1$ theo AM-GM)

Dấu = xảy ra khi $a=b=c=1$

1. Tương tự câu 2 ta chứng minh được:

$\sum \frac{ab}{c+1}\geq 6-2\sqrt{6}$

Mà $2(a+b+c)\geq 2\sqrt{3(ab+bc+ca)}=2\sqrt{6}$

Cộng 2 bất đẳng thức trên ta có đpcm




#624680 Inequalities From 2016 Mathematical Olympiads

Đã gửi bởi NTA1907 on 03-04-2016 - 22:24 trong Bất đẳng thức - Cực trị

Bài 6 (Hong Kong TST). Cho ba số thực dương $a,\,b,\,c$ thỏa mãn $abc=1.$ Tìm giá trị nhỏ nhất của

$$\frac{a^3+8}{a^3(b+c)}+\frac{b^3+8}{b^3(a+c)}+\frac{c^3+8}{c^3(b+a)}.$$

 

Không biết giải bài ở đây có vi phạm không anh nhỉ :( 
$VT=\sum \frac{a^3+1+1+6}{a^3(b+c)} \ge \sum \frac{3a+6}{a^3(b+c)}=\sum \frac{3(a+2)}{a^3(b+c)}$ 
Ta sẽ chứng minh $\sum \frac{3(a+2)}{a^3(b+c)} \ge \frac{27}{2}$ (*)
Chợt nhận thấy bài toán quen thuộc của IMO 1995  
Nếu $abc=1$ thì $\sum \frac{1}{a^3(b+c)} \ge \frac{3}{2}$ 
Áp dụng suy ra $\frac{6}{a^3(b+c)} \ge 9$ 
Lại có $\sum \frac{3}{a^2(b+c)}=\sum \frac{3(bc)^2}{b+c} \ge \frac{3(\sum ab)^2}{2\sum a}$ (1) 
Lại có $(\sum ab)^2 \ge 3.abc(a+b+c)$ nên từ (1) suy ra $\frac{3(\sum ab)^2}{2\sum a} \ge \frac{9}{2}$ 
Cộng lại suy ra (*) được chứng minh 
Vậy giá trị nhỏ nhất là $\frac{27}{2}$ khi và chỉ khi $a=b=c=1$

Cách khác

Ta có:
$\sum \frac{a^{3}+8}{a^{3}(b+c)}=\sum \frac{1}{b+c}+\sum \frac{8}{a^{3}(b+c)}\geq \frac{9}{2(a+b+c)}+\sum \frac{8(bc)^{2}}{a(b+c)}\geq \frac{9}{2(a+b+c)}+\frac{8(bc+ca+ab)^{2}}{2(ab+bc+ca)}=\frac{9}{2(a+b+c)}+(ab+bc+ca)+3(ab+bc+ca)\geq \frac{9}{2(a+b+c)}+\sqrt{3abc(a+b+c)}+3.3\sqrt[3]{(abc)^{2}}=\frac{9}{2(a+b+c)}+\frac{\sqrt{3(a+b+c)}}{2}+\frac{\sqrt{3(a+b+c)}}{2}+9\geq 3\sqrt[3]{\frac{27}{8}}+9=\frac{27}{2}$
Dấu = xảy ra khi và chỉ khi $a=b=c=1$



#624688 Inequalities From 2016 Mathematical Olympiads

Đã gửi bởi NTA1907 on 03-04-2016 - 22:36 trong Bất đẳng thức - Cực trị

Bài 8 (Selection Of Kiev To UMO). Với $a,\,b,\,c$ là ba số thực dương thỏa mãn $a+b+c=3.$ Chứng minh rằng

\[\frac{a^2}{a+b^2}+\frac{b^2}{b+c^2}+\frac{c^2}{c+a^2} \geqslant \frac{3}{2}.\]

Áp dụng AM-GM ta có:

$a-\frac{a^{2}}{a+b^{2}}=\frac{ab^{2}}{a+b^{2}}\leq \frac{ab^{2}}{2b\sqrt{a}}=\frac{b\sqrt{a}}{2}$

Tương tự cộng lại ta được:

$\sum \frac{a}{a+b^{2}}\geq 3-\frac{1}{2}(b\sqrt{a}+c\sqrt{b}+a\sqrt{c})\geq 3-\frac{1}{2}.\sqrt{(a+b+c)(ab+bc+ca)}\geq 3-\frac{1}{2}.\sqrt{(a+b+c).\frac{(a+b+c)^{2}}{3}}=3-\frac{3}{2}=\frac{3}{2}$

Dấu = xảy ra$\Leftrightarrow a=b=c=1$




#625270 Inequalities From 2016 Mathematical Olympiads

Đã gửi bởi NTA1907 on 05-04-2016 - 22:41 trong Bất đẳng thức - Cực trị

Bài 3 (Korea Winter Program Practice Test). Cho ba số thực không âm $x,\,y,\,z$ thỏa mãn

\[(x+y-1)^2+(y+z-1)^2+(z+x-1)^2=27.\]
Tìm giá trị lớn nhất của biểu thức $x^4+y^4+z^4.$

Bài này đề gốc là tìm cả min cả max chứ anh?

302f2d34349b033bf9410dd512ce36d3d439bdfe.jpg




#606298 $\boxed{\text{Chuyên Đề}}$ Phương trình vô tỉ - Hệ phương...

Đã gửi bởi NTA1907 on 31-12-2015 - 14:26 trong Phương trình, hệ phương trình và bất phương trình

8/     $\sqrt{3x^{2}+6x+7}+\sqrt{5x^{2}+10x+14}=4-2x-x^{2}$

$VT=\sqrt{3(x+1)^{2}+4}+\sqrt{5(x+1)^{2}+9}\geq \sqrt{4}+\sqrt{9}=5$

$VP=5-(x+1)^{2}\leq 5$

$VT=VP\Leftrightarrow x=-1$




#635674 Topic về phương trình và hệ phương trình

Đã gửi bởi NTA1907 on 26-05-2016 - 13:19 trong Phương trình - hệ phương trình - bất phương trình

Bài 445: 1) $x^3-6=\sqrt[3]{x+6}$

2)$\sqrt{5-x}-\sqrt{3x+1}=8x^2+16x-24$

3)$\sqrt{2x-1}-\sqrt{5x-2}=(5x-2)^3-(2x-1)^3$

4)$\sqrt[3]{x^2+1}+\sqrt[5]{2x^2+2}=\sqrt[3]{x+2}+\sqrt[5]{x+3}$

5)$x+\sqrt{2x}=\frac{1}{x}+\sqrt{x+\frac{1}{x}}$

1, Đặt $\sqrt[3]{x+6}=t$

Ta có hệ: $\left\{\begin{matrix} &t^{3}=x+6 \\ &x^{3}=t+6 \end{matrix}\right.$

Trừ 2 pt trên vế theo vế ta được:

$(t-x)(t^{2}+tx+x^{2}+1)=0$

$\Leftrightarrow t=x$

...

2, ĐK: $\frac{-1}{3}\leq x\leq 5$

Pt$\Leftrightarrow \frac{x-1}{\sqrt{5-x}+\sqrt{3x+1}}+2(x-1)(x+3)=0$

$\Leftrightarrow (x-1)\left ( \frac{1}{\sqrt{5-x}+\sqrt{3x+1}}+2(x+3) \right )=0$

$\Leftrightarrow x=1$(vì phần trong ngoặc luôn dương)

 

Các bài 3,4,5 đều là các bài tương tự như bài 2




#652285 Topic về phương trình và hệ phương trình

Đã gửi bởi NTA1907 on 01-09-2016 - 21:14 trong Phương trình - hệ phương trình - bất phương trình

Bài 499: $\left\{\begin{matrix} &x(y-9)+\sqrt{y-1}+1=0 \\ &y(18x^{2}+1)=3x+22+(x+1)^{2} \end{matrix}\right.$

Bài 500: $\left\{\begin{matrix} &(x-2y)\left ( 3x+8y+4\sqrt{x^{2}-4xy+4y^{2}-16} \right )=-6 \\ &(y-4x)\left ( 3y+2x+2\sqrt{x^{2}-4xy+4y^{2}-16} \right )=-10 \end{matrix}\right.$




#650662 Topic về phương trình và hệ phương trình

Đã gửi bởi NTA1907 on 21-08-2016 - 16:53 trong Phương trình - hệ phương trình - bất phương trình

Bài 483: Giải hệ phương trình:

$\left\{\begin{matrix} &\dfrac{2x^{2}+4y^{2}}{xy}=4\sqrt{(\dfrac{2}{y}-\dfrac{3}{x})(x+y)}-1 \\ &\sqrt{(x+1)^{2}+xy+3x+2y+5-2x\sqrt{x(y+3)}}=\sqrt{x}+\sqrt{y+3} \end{matrix}\right.$

Một lời giải khác...

Phương trình (1) tương đương:

$2x^{2}+xy+4y^{2}=4\sqrt{xy(2x-3y)(x+y)}$

$\Leftrightarrow (4y^{2}+4xy)+(2x^{2}-3xy)=2\sqrt{(2x^{2}-3xy)(4xy+4y^{2})}$

$\Leftrightarrow 2x^{2}-3xy=4y^{2}+4xy$

$\Leftrightarrow x=4y$ hoặc $y=-2x$(1)

Phương trình (2) tương đương:

$\sqrt{x^{2}-2x\sqrt{x(y+3)}+x(y+3)+2(x+y+3)}=\sqrt{x}+\sqrt{y+3}$

$\Leftrightarrow \sqrt{\left ( x-\sqrt{x(y+3)} \right )^{2}+2(x+y+3)}=\sqrt{x}+\sqrt{y+3}\geq \sqrt{2(x+y+3)}$

$\Rightarrow x+y+3+2\sqrt{x(y+3)}\geq 2(x+y+3)$

$\Leftrightarrow (\sqrt{x}-\sqrt{y+3})^{2}\leq 0$

$\Leftrightarrow x=y+3$(2)

Từ (1) và (2) ta giải được: $x=4, y=1$




#645382 Topic về phương trình và hệ phương trình

Đã gửi bởi NTA1907 on 18-07-2016 - 11:10 trong Phương trình - hệ phương trình - bất phương trình

Bài 462: Giải phương trình:

$\sqrt[4]{x^{4}+1}=\sqrt{x^{2}+3x+1}+\sqrt{2x+10}$




#634681 Topic về phương trình và hệ phương trình

Đã gửi bởi NTA1907 on 22-05-2016 - 12:45 trong Phương trình - hệ phương trình - bất phương trình

Đây là những bài tập chưa có lời giải trong Topic về phương trình và hệ phương trình, mong các bạn sớm hoàn thiện những bài tập này trước khi đăng bài mới để tránh loãng topic

 

Bài 230: $\left\{\begin{matrix} &x^{2015}+xy^{2014}=y^{4030}+y^{2016} \\ &7y^{4}+13x+8=2y^{4}.\sqrt[3]{x(3x^{2}+3y^{2}-1)} \end{matrix}\right.$

 

P/s: Những bài có đáp án sẽ được tô màu đỏ.

+) $y=0$ không là nghiệm của hệ

+) $y\neq 0$, chia cả 2 vế của pt(1) cho $y^{2015}$ ta có:

$\left ( \frac{x}{y} \right )^{2015}+\frac{x}{y}=y^{2015}+y$

$\Leftrightarrow \frac{x}{y}=y\Leftrightarrow x=y^{2}$

Thay $x=y^{2}$ vào pt(2) ta có:

$7x^{2}+13x+8=2x^{2}\sqrt[3]{x(3x^{2}+3x-1)}$

+) $x=0$ không là nghiệm của pt

+) $x\neq 0$, chia cả 2 vế của pt cho $x^{3}$ ta được:

$\frac{7}{x}+\frac{13}{x^{2}}+\frac{8}{x^{3}}=2\sqrt[3]{3+\frac{3}{x}-\frac{1}{x^{2}}}$

Đặt $a=\frac{1}{x}$

$\Rightarrow 8a^{3}+13a^{2}+7a=2\sqrt[3]{3+3a-a^{2}}$

Đặt $b=\sqrt[3]{3+3a-a^{2}}$

Khi đó ta có hệ: $\left\{\begin{matrix} &8a^{3}+13a^{2}+7a=2b \\ &-a^{2}+3a+3=b^{3} \end{matrix}\right.$

Cộng vế theo vế 2 pt trên ta được:

$(2a+1)^{3}+2(2a+1)=b^{3}+2b$

...




#634268 Topic về phương trình và hệ phương trình

Đã gửi bởi NTA1907 on 20-05-2016 - 13:04 trong Phương trình - hệ phương trình - bất phương trình

Bài 429: $\left ( \sqrt{\sqrt{x^{2}-8x+7}+\sqrt{x^{2}-8x-9}} \right )^{x}+\left ( \sqrt{\sqrt{x^{2}-8x+7}-\sqrt{x^{2}-8x-9}} \right )^{x}=2^{x+1}$

Bài 430: $\left\{\begin{matrix} &x+y+xy=z^{2^{2003}}+2z^{2^{2002}} & \\ &x^{4}+y^{4}=2z^{2^{2004}} & \\ &(x+y)^{z-1}=(z+2004)^{x-y} & \end{matrix}\right.$




#623822 Topic về phương trình và hệ phương trình

Đã gửi bởi NTA1907 on 31-03-2016 - 12:58 trong Phương trình - hệ phương trình - bất phương trình

Bài 355: $x^{3}+3x^{2}-4x+1=(x^{2}+3)\sqrt{x^{2}-x+1}$(Đề thi THPT chuyên Hùng Vương 2016)

Bài 356: $(2x+4)\sqrt{5-x^{2}}+(x-1)\sqrt{5+x^{2}}\leq 7x+5$(Đề thi THPT Trung Giã 2016)

Bài 357: $\left\{\begin{matrix} &(x+2)(x-y^{2}+1)+\sqrt{(x+1)(y^{2}+1)}=2y^{2}+3 \\ &5y^{2}+22=3\sqrt{x^{2}+8y^{2}}+\dfrac{18}{x+\sqrt{x^{2}-1}} \end{matrix}\right.$(Thi thử Toanhoc247 2016)

Bài 358: $\left\{\begin{matrix} &(x+y)(x^{2}+12y^{2}+x(y+1)+3)=12y^{2}-2y+5 \\ &5x+5+2(y-2)\sqrt{x^{2}+3}=y^{3}-4y^{2}+5y+\sqrt[3]{x^{2}+2y^{2}-4y+7} \end{matrix}\right.$(Thi thử Vinastudy lần 1 2016)

 




#609712 Topic về phương trình và hệ phương trình

Đã gửi bởi NTA1907 on 18-01-2016 - 22:01 trong Phương trình - hệ phương trình - bất phương trình

Bài 66: $(x^{2}-x)\sqrt{2x+1}\leq x^{3}-2x-1$ (trích bài của bạn nguyenhien2000)

ĐK: $x\geq \frac{-1}{2}$

Đặt $\sqrt{x+\frac{1}{2}}=t$

Khi đó ta có:

$(x^{2}-x)t\leq x^{3}-t^{2} \Leftrightarrow (x^{2}+t)(x-t)\geq 0$

Mà $x^{2}+t> 0$ nên $x\geq t$

$\Leftrightarrow \left\{\begin{matrix} &x\geq 0 \\ &x^{2}-2x-1\geq 0 \end{matrix}\right.$

$\Leftrightarrow x\geq 1+\sqrt{2}$




#608744 Topic về phương trình và hệ phương trình

Đã gửi bởi NTA1907 on 13-01-2016 - 13:03 trong Phương trình - hệ phương trình - bất phương trình

Bài 20: Giải phương trình:

a, $4x^{2}+(2x-5)\sqrt{4x+2}+17=(2x+3)\sqrt{6-4x}$
b, $\sqrt[3]{\frac{1}{3}-x^{2}}+\sqrt{x-\frac{2}{9}}=1$
 



#610738 Topic về phương trình và hệ phương trình

Đã gửi bởi NTA1907 on 24-01-2016 - 16:10 trong Phương trình - hệ phương trình - bất phương trình

Đây là những bài tập chưa có lời giải trong Topic về phương trình và hệ phương trình, mong các bạn sớm hoàn thiện những bài tập này trước khi đăng bài mới để tránh loãng topic

Bài 31: $3x^{3}-6x^{2}-3x-17=3\sqrt[3]{9(-3x^{2}+21x+5)}$

Pt$\Leftrightarrow (3x-3)^{3}+27(3x-3)=9(-3x^{2}+21x+5)+27\sqrt[3]{9(-3x^{2}+21x+5)}$

$\Rightarrow 3x-3=\sqrt[3]{9(-3x^{2}+21x+5)}$

$\Leftrightarrow 3x^{3}-6x^{2}-12x-8=0$

...




#611470 Topic về phương trình và hệ phương trình

Đã gửi bởi NTA1907 on 28-01-2016 - 13:17 trong Phương trình - hệ phương trình - bất phương trình

Đây là những bài tập chưa có lời giải trong Topic về phương trình và hệ phương trình, mong các bạn sớm hoàn thiện những bài tập này trước khi đăng bài mới để tránh loãng topic

Bài 133: $\left\{\begin{matrix} &x^{5}-y^{4}+2x^{2}y=2 \\ &y^{5}-x^{4}+2xy^{2}=2 \end{matrix}\right.$

Lấy 2 pt trừ cho nhau ta dc:

$(x^{5}-y^{5})+(x^{4}-y^{4})+2xy(x-y)=0$

$\Leftrightarrow (x-y)(x^{4}+x^{3}y+x^{2}y^{2}+xy^{3}+y^{4})+(x-y)(x^{3}+xy^{2}+x^{2}y+y^{3})+2xy(x-y)=0$

$\Leftrightarrow (x-y)(x^{4}+x^{3}y+x^{2}y^{2}+xy^{3}+y^{4}+x^{3}+x^{2}y+xy^{2}+y^{3}+2xy)=0$

TH1: $x=y$

Thay vào ta dc:

$x^{5}-x^{4}+2x^{3}=2$

$\Leftrightarrow (x-1)(x^{4}+2x^{2}+2x+2)=0 \Rightarrow x=y=1$

TH2: $x^{4}+x^{3}y+x^{2}y^{2}+xy^{3}+y^{4}+x^{3}+x^{2}y+xy^{2}+y^{3}+2xy=0$

Ai giải quyết được TH này ko?




#621142 Topic về phương trình và hệ phương trình

Đã gửi bởi NTA1907 on 19-03-2016 - 13:46 trong Phương trình - hệ phương trình - bất phương trình

Bài 326: $\begin{cases} & x+\sqrt{x(x^{2}-3x+3)}=\sqrt[3]{y+2}+\sqrt{y+3}+1 \\ & 3\sqrt{x-1}-\sqrt{x^{2}-6x+6}= \sqrt[3]{y+2}+1 \end{cases}$

ĐK: $x\geq 3+\sqrt{3}, y\geq -3$
Pt(1)$\Leftrightarrow x-1+\sqrt{(x-1)^{3}+1}=\sqrt[3]{y+2}+\sqrt{y+3}$
Đặt $x-1=a, \sqrt[3]{y+2}=b$
$\Rightarrow a+\sqrt{a^{3}+1}=b+\sqrt{b^{3}+1}$
$\Rightarrow a=b$
Dễ rồi



#612559 Topic về phương trình và hệ phương trình

Đã gửi bởi NTA1907 on 02-02-2016 - 22:01 trong Phương trình - hệ phương trình - bất phương trình

Câu 179: Giải PT: $\sqrt{\sqrt{3}-x}=x\sqrt{\sqrt{3}+x}$

ĐK: $-\sqrt{3}\leq x\leq \sqrt{3}$

Pt$\Rightarrow \sqrt{3}-x=x^{2}(\sqrt{3}+x)$

$\Leftrightarrow x^{3}+\sqrt{3}x^{2}+x-\sqrt{3}=0$

$\Leftrightarrow (x+\frac{1}{\sqrt{3}})^{3}=\frac{10\sqrt{3}}{9}$

$\Leftrightarrow x=\sqrt[3]{\frac{10\sqrt{3}}{9}}-\frac{1}{\sqrt{3}}$(TM)




#626649 Topic về phương trình và hệ phương trình

Đã gửi bởi NTA1907 on 11-04-2016 - 21:28 trong Phương trình - hệ phương trình - bất phương trình

Bài 386: $\left\{\begin{matrix} &x-2\sqrt{x^{2}-2x+4}=y+1-2\sqrt{y^{2}+3} \\ &\sqrt{4x^{2}+x+6}-5\sqrt{y+2}=\sqrt{xy-2y-x+2}-1-2y-\left | x-2 \right | \end{matrix}\right.$

Lời giải đầy đủ đây các bạn tham khảo

12715786_466729563510601_4731407955057557081_n.jpg